4
$\begingroup$

$\DeclareMathOperator\GL{GL}$Let $G \subset M_{n\times n~}(\mathbb Z)$ be a finitely generated subgroup of $\GL(n,\mathbb Q)$ (i.e. $g\in G$ is an invertible matrix with entries in $\mathbb Z$). Then $G$ acts on $\mathbb R^n = \mathbb Z^n \otimes_{\mathbb Z} \mathbb R$ through $\GL(n,\mathbb Q)$.

Suppose that there is a rational affine subspace $V \subset \mathbb R^n$ (by this, I mean that there is a sub-lattice $L \subset \mathbb Z^n$ and $a \in \mathbb Z^n$ such that $V = a + (L \otimes_{\mathbb Z} \mathbb R)$), and $V$ is invariant under the action of $G$ (i.e. for any $v\in V, g\in G$, we have $g\cdot v \in V$). Moreover, there exists $v \in L$ (in fact, we can take $v=a$) such that $$G \cdot v = L.$$

Question: is there a bounded subset $P \subset V$ such that $$\bigcup_{g \in G}\ g\cdot P = V \quad ? $$

Any suggestion on relevant questions/references is very welcome! Particularly, I don't know which field studies such problems ….

Edit:

Example. Consider $(0,1)+L:=(\mathbb Z,1) \subset \mathbb Z^2$, and $$G=\{\begin{pmatrix} 1&k\\0&1\end{pmatrix}\mid k\in\mathbb Z\}.$$ For $v=(0,1)$, we have $G \cdot v =(0,1)+L$. In this case, we can take $P$ to be the interval from $(0,1)$ to $(1,1)$.

$\endgroup$
13
  • $\begingroup$ Usually a fundamental domain is required to satisfy stricter conditions than your $P$. You only need $P$ bounded? $\endgroup$
    – LSpice
    Jan 9, 2021 at 3:14
  • 1
    $\begingroup$ Yes, for me "$P$ bounded" is enough. I don't know how to characterize such set in one word in the title, so I coin the name "fundamental domain". $\endgroup$
    – Li Yutong
    Jan 9, 2021 at 3:22
  • 3
    $\begingroup$ The hypothesis cannot be satisfied by any subgroup of ${\rm GL}_n({\bf Z})$, finitely generated or not, because the action of ${\rm GL}_n({\bf Z})$ preserves the subgroups $M {\bf Z}^n$ of ${\bf Z}^n$, so for example cannot take a nonzero vector $v$ to $2v$, let alone to the zero vector. Did you mean to require that $G \cdot v$ consist of all primitive integer vectors? $\endgroup$ Jan 9, 2021 at 3:41
  • 1
    $\begingroup$ You are right! I should say "$\mathbb Z$-linear combinations of the elements in $G \cdot v$ is equal to $\mathbb Z^n$" $\endgroup$
    – Li Yutong
    Jan 9, 2021 at 4:46
  • 1
    $\begingroup$ Then the answer is negative, you can take for instance the standard linear representation of the permutation group on $n$ symbols. $\endgroup$ Jan 9, 2021 at 5:26

1 Answer 1

1
$\begingroup$

After reading the comments, I think the underlying algebraic question is: " if $G=U$ is a unipotent linear algebraic group defined over $\mathbb{Q}$, then is the arithmetic unipotent subgroup $U(\mathbb{Z})$ cocompact in $U(\mathbb{R})$? "

The answer to the above question is Yes, as well known to students in geometry of numbers, reduction theory of quadratic forms, and arithmetic groups. For example, a proof is contained in Borel/Harish-Chandra, S 6.10. (See image below).

E.g. the integral Heisenberg group $H(\mathbb{Z})$ is cocompact in $H(\mathbb{R})$.

If the OP does not study the full arithmetic unipotent $U(\mathbb{Z})$, but restricts to a subgroup $G \subset U(\mathbb{Z})$, then the answer to the above question is No unless $G$ is finite index in $U(\mathbb{Z})$. Indeed it's evident that discrete subgroups $L'$ of $\mathbb{Z}^n$ are cocompact in $\mathbb{R}^n$ if and only if $[L':\mathbb{Z}^n]<+\infty$.

enter image description here

$\endgroup$
1
  • 1
    $\begingroup$ It is unclear to me that this is the intended question. But the question was rewritten so many times (and will be probably rewritten again) that it's hard to tell. $\endgroup$ Jan 10, 2021 at 18:00

Your Answer

By clicking “Post Your Answer”, you agree to our terms of service and acknowledge you have read our privacy policy.

Not the answer you're looking for? Browse other questions tagged or ask your own question.